« first day (36 days earlier)      last day (538 days later) » 
00:00 - 16:0016:00 - 00:00

12:00 AM
0
Q: Is $\int_{-R}^R(t+\frac{ia}{2})^2e^{-t^2}dt=(\frac{1}{2}-\frac{a^2}{4})\int_{-R}^R e^{-t^2}dt-Re^{-R^2}$?

integralLet $t\in [-R,R], a\in\mathbb{R}$. I want to know if the following equation holds: $$\int_{-R}^R(t+\frac{ia}{2})^2e^{-t^2}dt=(\frac{1}{2}-\frac{a^2}{4})\int_{-R}^R e^{-t^2}dt-Re^{-R^2}.$$ If I start with left hand side: $\int_{-R}^R(t+\frac{ia}{2})^2e^{-t^2}dt =\int_{-R}^R t^2e^{-t^2}dt+ia\int_{-...

0
Q: Finding the nth term for .19, .1919, .191919, ...

MathingHaving trouble finding a recursive defintion for the following: n(0)=.19, n(1)=.1919, n(2)=.191919, n(3)=.19191919

0
Q: How to find the basis for this sub-space?

Chadhow to find a basis of this sub-space? and what is its dimension? note that every matrix in the basis has to have rank of 1. enter image description here

0
Q: Prove f is strictly monotone -- cardinality involved

samsoniteLet $D \subset R$ and let $f: D\rightarrow R.$ Assume that #D $\geq 4$. Assume that $f$ is strictly 4-monotone, i.e., assume, for all $S\subset D, $ that [#$S=4] \rightarrow [(f|S)$ is strictly monotone]. Prove that f is strictly monotone. I have been able to prove monotone included problems, bu...

Questions tend to get more attention when they have a tag for a broad area of mathematics relevant to the question. Some of these tags might fit. (from a bot)Normal Human 21 secs ago
0
Q: Matrix norm inequality proof: inverse of two p.s.d matrices sum

YoooHanI wonder if the following matrix norm inequality holds: Let $A$ and $B$ are both strictly symmetric positive definite matrix $\|(A+B)^{-1}\|_2\leq \|A^{-1}\|_2$ ? Thanks in advance.

0
Q: What does it mean to 'show that' coefficients are a solution of this system of linear equations?

benxyzzyI don't really know what counts as a proof and haven't been taught maths since 16yo (29yo PhD now). I've got working knowledge of e.g. basic linear algebra, geometry, and statistics, but this feels like running before I can walk so I'm now going back and filling in the elementary gaps. I'm workin...

0
Q: $ax^2+bx+c=(rx+s)^2$ implies $b^2-4ac=0$?

NumThcuriousIf a quadratic equation with integral coefficients is :$$ax^2+bx+c=(rx+s)^2$$ Can we say that the discriminant $$b^2-4ac=0?$$ If so how do you prove it?

Short title. Short question. Tagged proof-verification. $ax^2+bx+c=(rx+s)^2$ implies $b^2-4ac=0$?
 
12:32 AM
0
Q: Check best answer with the most views

BolboaIs there a way to see which of one of a user's answers reached the most people? I know this is possible for questions, since a user's questions can be ordered based on their view count, however, I was wondering if the same can be done for answers. Can they be ordered based on the number of view...

 
0
Q: Number Theory: Show that $10^{3^n}\equiv 1\pmod{3^{n+2}}$ but $3^{n+3}\not\mid 10^{3^n}-1$

MathQuestionShow that for all $n\in\mathbb{N}$, $10^{3^n}\equiv 1\pmod{3^{n+2}}$ but $3^{n+3}\not\mid 10^{3^n}-1$. I think I've proved this problem, but I was unsure if my proof was correct: Proof Let $n=1$. Then, $10^3=1000\equiv1\pmod{3^3}$ since $3^2=9\mid999$ and $3\mid111\implies 3^3\mid999$; but, $3...

0
Q: A chi-squared fit of a general quadratic polynomial is done to ten data points. What is the number of degrees of freedom of this fit?

BLAZEI think the correct answer is $7$ because the general quadratic is $$y_i=ax_i^2 + bx_i + c$$ Using the formula $$\color{red}{\fbox{Number of degrees of freedom = Number of data points - Number of Parameters}}$$ The $3$ parameters are $y_i,x_i^2,x_i$ So $10-3=7$ degrees of freedom. The correct...

0
Q: Finding Recursive Definition for the following:

MathingHow would i start off to find a recursive definition for x(1)=.19 x(2)=.1919 x(3)=.191919 ... x(n+1)= x(0) + (something goes here, but what)

0
Q: Show that $x^2-\frac{2}{x}$ is less than 3 for all x

Dean BearlHow do I show that $x^2-\frac{2}{x}$, without looking at a graph? I am open to algebraic and calculus related answers.

0
Q: Matrix form of Fourier and Fresnel Transform

M AtrixI am wondering how to write up the matrix form of, say, Fresnel or Fourier transform. I know that for the case of Fresnel it would be Toeplitz matrix.

0
Q: WKB Approx. Exercise for 2nd Order ODE

user145047Working with the following ODE: $y'' - \frac{1}{2}y' + (\frac{1}{16} + x - e^x)y=0$ W.T.S. there are solutions of the form: $y_1 = ${1+O$(xe^{-x/2})$}$exp(-2e^{x/2})$ and $y_2 = ${1+o$(1)$}$exp(2e^{x/2})$ as $x \rightarrow \infty$ My Attempt Thm1: $\frac{d^2y}{dx^2} + f(x)\frac{dy}{dx}+g...

Title contains exercise. WKB Approx. Exercise for 2nd Order ODE
 
12:55 AM
0
Q: Rename [watch-os-...] tags to [watchos-...]?

stkentCurrently, the following three tags exist and pertain to the operating system running on Apple Watches: watch-os watch-os-2 watch-os-2-beta3 However, this naming is inconsistent with other tags for Apple platforms. For example: "iOS" is mapped to ios, ios7, ios-simulator, etc. "tvOS" is map...

 
0
Q: Prove that if a function is uniformly continuous then its bounded

Maths beginnerSuppose f:[0,1)->R is uniformly continuous on [0,1). Prove that the function is bounded (i.e. that range(f) is a bounded set.

0
Q: Smallest symmetric group with subgroup Q

J. DoeWhat is the smallest $n$ such that the quaternion group is a subgroup of $S_n$?

0
Q: Kraft's lemma and Shannon

user296836I'm stuck on a problem and just need some help. If p=(3/4, 1/4) I'd like to construct a tree for pp and pp*p using the construction in Shannon's theorem. I'm completely stuck since I don't understand his construction. I'd really appreciate some help/

0
Q: Isolate x in the exponential decay equation?

MikeHow would I algebraically isolate x in the following exponential decay equation, so that x is most easily derived? $$ y = A e^{-\lambda x} \quad $$

0
Q: Examples of nice theory

LevitanI have a language $L=\{P\}$, $P$ is binary predicate symbol. There is an implementation $N$ of language $L$, where carrier is $\mathbb N=\{0,1,2,\dots\}$ and $P_N=\{(a,b) \in \mathbb N^2: a \leq b\}$ I call a theory $T$ of language $L$ "nice" iff $N |= T$ and $N$ is an inducted subimplementatio...

Words such as nice do not add information to titles. Please edit the title so that it better describes the specifics of your question. Do not hesitate to make it longer or include a formula if needed. More tips here. (from a bot)Normal Human 20 secs ago
0
Q: Is a singular point a closed interval?

Alvin NunezFor example, is {0} considered a closed interval? Why or why not? Doesn't it contain all (it's only) limit point of 0?

0
Q: If $A\ge 0$ and $A^k>0$ for some positive integer $k$ then $\rho (A) > 0$?

H.SWe say $A \ge 0$ if all $a_{ij}\ge 0$. Now let $A\ge 0$ and $A^k>0$ for some positive integer $k$. Why does $\rho (A) > 0$?

0
Q: Is this sequence increasing?

ClaycrusherDefine $(p_n)_n$ recursively by $p_0(x) = 0$ and $p_{n+1}(x) = p_n(x) + \frac{1}{2}(x - p_n(x)^2)$. I'm pretty sure this sequence is increasing, but I want to prove it. I've tried using induction, but I get stuck: (Assume $p_k \leq p_{k+1}$) $p_{k+1} \leq p_{k+2}$ $\Rightarrow p_{...

0
Q: The weird transition process

The HomeworkerLet $X_t$ be distributed according to $\text{Poisson}(\lambda_1)$ where $\lambda_1$ is the intensity parameter of the distribution and indepdently from this we have a process $Y_t$ distributed according to $\text{Poisson}(\lambda_2).$ Additionally we define a process $Z_t \in \{0,1\}$ such that ...

Short title. Title contains weird. The weird transition process
0
Q: Cross Validation of Least Squares

user296769If $y=Ax+\epsilon$, where $y$ is a vector of dimension $m$, $A$ is a matrix $m \times n$ and is skinny and full rank, and $x(\mu)$ is a vector of dimension $n$. Regularized least squares estimation of x with the regularization parameter $\mu$ is the vector $\hat x(\mu)$ that minimizes $||Ax-y||^2...

Questions tend to get more attention when they have a tag for a broad area of mathematics relevant to the question. Some of these tags might fit. (autocomment)Normal Human 20 secs ago
 
1:32 AM
0
Q: Discrimination against PDE questions?

grayQuantJust going through the questions tagged partial-differential-equations, I saw that many of them are unanswered and have zero upvotes. Is there a good reason why these types of questions are kind of ignored by the community?

 
0
Q: Please help showing a basic property of a (non necessarily distributive) lattice

user98231Suppose we have a lattice $L$ with the property that : $\forall x,y \in L$, $x \leq y$ implies $x \leftrightarrow y$ (where $x \leftrightarrow y$ means $x$ and $y$ are compatible - that is, the lattice generated by $x$ and $y$ is distributive). Now suppose $a, b \in L$ and $a < b'$, (where $...

0
Q: convex-concave lagrangian

queenceCan someone help me out? What makes the Lagrangian of an optimization problem "convex-concave"? In other words, what can I pick out from the Lagrangian that will tell me whether or not a function is strictly convex-concave and/or globally convex-concave? (I am specifically looking at a question...

0
Q: compute $E(1_{W\leq t}|X)$

Johnlet $(X,Y)$ independent random variables, $X>0$ and $W = XY$. how can i compute $E(1_{W\leq t}|X)$ using the distribution function of $Y$ ? some help would be appreciated

Short title. Short question. compute $E(1_{W\leq t}|X)$
0
Q: Two continuous functions intersection

Maths beginnerSuppose f,g:[a,b]->R are continous functions such that f(a)<=g(a) and f(b)>=g(b). Prove that f(c)=g(c) I know that I need to use IVT but I don't know exactly how

0
Q: Find when when one circle intersects another circle while rotating along the perimeter of another circle

Hans Petersondiagram If circle B is rotating along the perimeter of circle A, when will circle B intersect circle C? theline going from hA,hK to perimeter where hA,hK = (0,0) is y = (sqrt(rA^2 - (xA^2))/xA)x (xA,yA) will be on perimeter of circle A and the point (xd,yd) that is rB distance from (xA,yA) o...

Welcome to Math.SE, Hans Peterson. This site uses MathJax formatting of formulas. More tips here. (from a bot)Normal Human 21 secs ago
0
Q: Prove your list is complete

user253595I am struggling with this problem. Do I need to look at the cyclic group or subgroups? How do I prove the list is complete? List all f $\epsilon$ {Z}_6 such that f^2 = f. Prove that your list is complete.

Short title. Tagged proof-writing. Prove your list is complete
Welcome to Math.SE, Spenser Stone. Please don't use (self-learning) tag just because you were self-studying. This tag is only for questions about the process of self-studying. (from a bot)Normal Human 21 secs ago
0
Q: finding a recursive definition for .191919...

Mathingwould one say that this is the recursive definition for a repeating decimal like .19191919..... Base Case: X(0) := .19 X(1) := .1919 X(2) := .191919 ... X(n) := X(0) + .19*((10^-2)^n) recursive step: X(n+1)= X(0) + .19((10^-2)^(n+1))

This site uses MathJax formatting of formulas. More tips here. (autocomment)Normal Human 21 secs ago
0
Q: T-estimator is T-Distibuted

jessicaI am trying to prove that $T_n=\frac{\bar{X}_n - \mu}{S/\sqrt{n}}\sim t_{n-1}$. How do you show this? Is the T-distrbution a ratio between a standard normal and a Chi?

Short title. Short question. T-estimator is T-Distibuted
0
Q: Prove that if det(A) = 0, then there exists a non-zero vector v such that Av = 0

JoeProve that if det(A) = 0, then there exists a non-zero vector v such that Av = 0, and that there must exist at least one eigenvector corresponding to each distinct eigenvalue of a square matrix.

0
Q: How to prove the recurrence relation for this generating function problem?

hlyatesI am a software engineer and I am learning combinatorics theory on my own. I recently got stumped by the following problem. Problem Let $a_{0} = 2$ where $a_{n} = 3a_{n-1} - 2$ with $n>0$. Given $G(x) = \sum_1^na_{k}x^k $ be the generating function. Prove $G(x) = \frac{2-4x}{(1-3)(1-x)}$ Att...

 
2:20 AM
0
Q: Hopf Algebra Identity

IanI am reading Kassel's book entitled "Quantum Groups". Let H be a Hopf Algebra and let $\epsilon$ and $S$ be the co-unit and antipode, respectively. On page 52 he has the formula $\epsilon\left(\sum_{(x)}\epsilon(x')S(x'')\right)=\epsilon(\eta\epsilon(x))$ where the sum uses Sweedler's conventio...

Welcome to Math.SE, Ian. Questions tend to get more attention when they have a tag for a broad area of mathematics relevant to the question. Some of these tags might fit. (from a bot)Normal Human 21 secs ago
 
0
Q: ability to see why my question was voted down

AlexOK, so my question was both up-voted and down-voted, I do not dispute the fact that there may have been a good reason for it to be down-voted. but can a feature be implemented that forces down-voters to give a short statement as to why they decided that, and show the person who asked the question...

 
0
Q: Matrix Inversion distribution

user296769How do you distribute the inversion in $(A^TA+\lambda I)^{-1}A^Ty$ assuming $A$ is a $n \times n$ square invertible matrix, $y$ is a vector with the dimension of $n$, and $\lambda$ is a constant?

0
Q: $\lim_{x\to\infty} cln(x) - x$

user296856$$L = \lim_{x\to\infty} cln(x) - x$$ Does L diverge to $-\infty$ for all positive c? It seems so for very large c, but taking the limit as c goes to infinity yields $$\lim_{x\to\infty} xln(x) - x = \infty$$ According to wolfram alpha. Is it wrong to set c = x?

Welcome to Math.SE, user296856. A title should not be all-MathJax; having some plain text helps with search and navigation. (from a bot)Normal Human 21 secs ago
0
Q: Help with Trig Sub

Pareod$\int_\infty^\infty \{\frac{x^2}{x^4+1}\}dx$ Firstly, the bounds are -infinity to infinity, but I'm not too familiar with MathJax. Secondly, I'm trying to understand trig sub better, because I never could get a good handle on it. All I know is that this integral is supposed to reduce to the in...

0
Q: Understanding mathematical notation for coding problems.

ApathyBearThe majority of my questions revolve around code (thus my activity on stackoverflow), but I've been going over interview question that assume a good understanding of mathematical notation. I don't have a great mathematical background, so I am having trouble understanding what the question is act...

Welcome to Math.SE, Pareod. Words such as help do not add information to titles. Please edit the title so that it better describes the specifics of your question. Do not hesitate to make it longer or include a formula if needed. More tips here. (autocomment)Normal Human 28 secs ago
0
Q: Can someone explain the division in this proof of the sum of harmonic sequence? (n+1)h(n) - n?

smitty_werbermanjensenSo... this is the explanation my instructor gives in his PDF, but I can't make heads or tails of it. Use mathematical induction to prove that for all positive integers n: H1 + H2 + . . . + Hn = (n + 1)Hn − n. solution: The base case is easy. For the induction step we assume H1+H2+. . .+Hk = (...

This site uses MathJax formatting of formulas. More tips here. (from a bot)Normal Human 21 secs ago
0
Q: Mathematical statistics analyzing excel

Marissa FarinaA realtor is trying to determine the relationship between a home's price and its square footage. He runs a linear regression in order to predict the value of the home's list price based on its square footage. Based on the regression output below, identify the following: The slope is 35.03637 a...

This site uses MathJax formatting of formulas. More tips here. (from a bot)Normal Human 21 secs ago
 
2:53 AM
0
Q: how to logout? (I think this title is self explanatory even if short, forgive me if I am wrong)

roy smiththere is no visible (to me) button for logging out. how to do so? this is an attempt to comply with Frances Dorais' request.

 
3:03 AM
0
Q: Prove that a product of two summable sequences is summable

user296855Definition of a Mash-up product Prove that a "mash-up" of two summable sequences is summable. (notice the sequence does not necessarily have to be non-negative)

0
Q: Stochastic Problem

CtxGiven the following situation: 99.9 percent of people have read a certain book. 99.7 percent of the people, who have read the book, can answer a certain question about it. Of these, who did not read the book, 0.04% answer the question correctly by chance. How big is the probability, that you have...

This site uses MathJax formatting of formulas. Consider adding a tag for a broader subject area to which the question belongs. Some of these tags might fit. More tips here. (autocomment)Normal Human 21 secs ago
0
Q: How do I find decimal powers?

user296861I have no idea how to do this. For example, how does one compute 10 raised to 0.90 without a calculator?

Short title. Short question. How do I find decimal powers?
0
Q: Recurrence Relations:

OnikouzouI have this recurrence relation I'm supposed to do, and I can't seem to figure out the next step: $T_{n} = T_{n-1} + 2n^2, T_{1} = 1$ What I have so far is this: $T_{n} = T_{n-1} + 2n^2$ $T_{n-2} + 2(n-1) + 2n^2$ <--- Is this right?

0
Q: Show that if $f(z)=\sum\limits_{n=0}^{\infty}a_n\para{z-a}^n$ on $B(a,R)$, then $f'(z)=\sum\limits_{n=0}^{\infty}na_n\para{z-a}^{n-1}$ on $B(a,R)$.

Tony S.F.What I have so far is using the Cauchy Integral Formula $f'(z)=\frac{n!}{2\pi i}\int_{B(a,R)}\sum\limits_{n=0}^{\infty}a_n(\zeta - a)^n / (\zeta - a)^2 d\zeta=\frac{n!}{2\pi i}\int_{B(a,R)}\sum\limits_{n=0}^{\infty}a_n(\zeta - a)^{n-2}d\zeta$. Not really sure where to go from here?

0
Q: General question about Direct Proofs

Random1144I was wondering what exactly constitutes a direct proof. For example if I prove an equation for some number > than n, with a formula given to be used such as Euler's Formula, would this still be considered a direct proof as it is not induction or contradiction, etc? Thanks

Words such as question are uninformative in titles. Please edit the title so that it better describes the specifics of your question. Do not hesitate to make it longer or include a formula if needed. More tips here. (from a bot)Normal Human 21 secs ago
0
Q: reference request - convergence of interval in the sense of Hausdorff distance

tankonetooneFor the definition of Hausdorff distance, please see here Suppose I have a sequence of interval $I_n=[a_n,b_n]\subset [0,1]$, then I read a in a paper it says that, up to a subsequence, $I_n\to I$ in the sense of Hausdorff distance, where $I=[a,b]$ and $a=\lim_n a_n$ and $b=\lim_n b_n$. The pap...

0
Q: Problem of the collector

okgoQuestion is below: Let $(X_{n})_{n≥1}$ be a sequence independent random variables uniformly distributed on {1, .., n}. Let $t_{n} = inf$ {m≥1 : {$X_{1}, ..., X_{m}$}= {1, ..., n}} be the first time for which all values have been observed. Let $t_{n}^{(k)} = inf${m≥1: |{$X_{1}, ..., ...

Short title. Title contains problem. Problem of the collector
 
3:41 AM
-3
Q: Downvoting my correct answer - Who or Why?

sreedevi sMy answer is correct (completely confident about that) and I am the first one to answer. Even then someone downvoted my answer. Someone added a comment below my answer that is exactly the summary of my answer. Can I find out who is downvoting or why?

 
3:52 AM
0
Q: Need help answer checking combinations and permutations problem

XariI was doing some problems for my quiz earlier today (which is now concluded) and went through some combination problems I'm unsure I answered correctly. If I'm wrong, can someone please explain why to me. Letters = {a, b, c} Create a string of length 25. 1) String must have exactly 9 A's. My a...

Title contains help, problem. Question contains please. Need help answer checking combinations and permutations problem
0
Q: A uniformly continuous function is bounded

user296855Suppose that the function f:[0,1) -> is uniformly continuous on [0,1). Prove that the function f is bounded. (i.e. that the range(f) is a bounded set)

0
Q: Complex differential equations involving $\bar{f}$ and $f'$

roger buckleyI am trying to prove that given a holomorphic function $f$, $u(x, y)=|f(x+iy)|$, $F=u^2$, we have $$\frac{\partial u}{\partial x}=\frac{\text{Re} (\bar{f}f')}{|f|}$$ $$\frac{\partial u}{\partial y}=-\frac{\text{Im}(\bar{f}f')}{|f|}$$ and $$\frac{\partial^2F}{\partial x^2}+\frac{\partial^2F}{\p...

Tagged differential-equations but mentions "partial". Complex differential equations involving $\bar{f}$ and $f'$
0
Q: Modes of Convergence, Real Analysis ch 2 problem 36

Morgan WeissIf $\mu(E_n) < \infty$ for $n\in\mathbb{N}$ and $1_{E_n}\rightarrow f$ in $L^{1}$, then $f$ is (a.e. equal to) the characteristic function of a measurable set. I am not sure how to define $E_n$ in order to prove this, any suggestions is greatly appreciated.

0
Q: Trying to answer this counting question

CoheenIn a ballroom dance class, participants are divided into couples for each drill session. One partner leads and the other follows for three minutes, and then the couple switches roles for the next three minutes. (a) Only four people show up on time. How many ways are there to pair them up? My ans...

Words such as question are uninformative in titles. Please edit the title so that it better describes the specifics of your question. Do not hesitate to make it longer or include a formula if needed. More tips here. (autocomment)Normal Human 20 secs ago
0
Q: Borel Sets sigma algebra

wolfNeed to connect Borel sets and sigma algebra. Also, need to show that a lebesgue measure contains all of the Borel Sets. I know that borel sets are the smallest open sigma algebra sets but I need to further my understanding and was having trouble with this.

0
Q: question quadratic-trigo equation

Archis Welankar$x^2-4x-cosx=-8$ is the equation and we want the number of solutions . Now i tried taking cos as constant but by formula for root we get a trig equation which cant be solved . Any help!thanks!

Words such as question are uninformative in titles. Please edit the title so that it better describes the specifics of your question. Do not hesitate to make it longer or include a formula if needed. More tips here. (from a bot)Normal Human 20 secs ago
0
Q: How to find a vector that is orthogonal to vector (u) in the plane of the two vectors (u) and (v)?

JosephIn my case, u is 1,-1,0 and v is 1,0,2. I know a vector orthogonal to u will be a+(-b)=0 but I don't know how to complete the problem.

 
4:21 AM
0
Q: Updating the question to add answer

GNKeshavaCheck this question for example. The last edit that OP did adds some answer or his own explanation for his question. Whether such edit is OK?

 
0
Q: Cramer-Rao Uniform Distribution

jessicaIf my data $X_i\sim U(0,\theta)$ is iid. What is the Cramer-Roa lower bound for a variance estimator such as the sample variance? $ S_n= \frac{1}{n} \sum_{i=1}^n (X_i-\bar{X})^2$ I am stuck because the uniform density $f(x)=1/\theta$, which means when I take the log likelihood I don't get any...

0
Q: Bezier Curves: Joining multiple vs Adding control points?

mavaviljI'm tring to figure out how to create "complicated" Bezier curves. E.g. a curve similar to $\frac{sin(x)}{x}$ for $x>0$. Is this achievable with the "typical" Bezier definition by adding control points or by joining multiple Bezier curves?

Questions tend to get more attention when they have a tag for a broad area of mathematics relevant to the question. Some of these tags might fit. (autocomment)Normal Human 20 secs ago
0
Q: Statistics P-Value Z-Test Formula?

Temp JonWhat is the formula for finding P-value for a 1-tailed Z-test. Is it P = 1 - Φ(Z) Or is it P = Φ(Z)?

0
Q: Prove number of injective functions from a set with $n$ elements to itself is n!

futuremathresearcherCan you please provide detailed explanation? Step by step analysis would be highly appreciated.

Short question. Question contains please, step by step. Prove number of injective functions from a set with $n$ elements to itself is n!
 
4:48 AM
0
Q: prove that 2^(n+100) = O(2^n)

al exxHaving trouble proving this for Big Oh. just dont know where to start with this one. I learn the basics over big oh notation so for this one, would i have so if s(n) = 2^(n+100) and (a(n)) = 2^n , then would i have to use the limit as n -> infiniti that (s(n) / a(n)) = 0?

0
Q: Triple integral over an edge of a cylinder

Pavlo ProtasenyaYes, I have been looking through this stackexchange site a lot, and have found a lot of things to assist me. But I still could not solve my problem. So my initial task is to : ( ∫ ∫ ) S ⁢ y z ⁢ dx ⁢ dy ...

Welcome to Math.SE, Pavlo Protasenya. This site uses MathJax formatting of formulas. More tips here. (autocomment)Normal Human 21 secs ago
0
Q: Find the power series for the function

JadeHow do I find the power series for f(x)=$\frac{x^3}{x+3}$ ? That to in summation notation.

0
Q: Correlation Coefficient

mathloverI am trying to understand the following equation for Correlation Coefficient: $r = \frac{\sum_{i=1}^{n}(x_i-\bar x)(y_i-\bar y)}{\sqrt(\sum_{i=1}^{n}(x_i - \bar x)^2\sum_{i=1}^{n}(y_i-y)^2)}$ Can someone dissect this equation and provide reasoning as to why this equation does what it does, ...

0
Q: How to solve differential equation $y'+\frac xy=x^2y^3$

ybtang21cI've tried to let $u=y^2$, and got $$u'+2x=2x^2u^2,$$ but I still can't solve it.

0
Q: Show solution has no maximum

MmmathI need to show that the solution of $ x'=t^3(x^2+1)+e^t$ cannot have a maxima. $\:$ First I tried to solve the equation, but to no avail.$\:$ Then I tried to show the solution is alway increasing, but I don't think it is. $\:$ I have no idea how to begin this problem. $\:$ If anyone could please ...

Short title. Question contains please. Show solution has no maximum
0
Q: Prove that {X ∈ P(Z)| X is finite} is enumerable.

DoryMotoryI am not sure how to approach this problem. if you could help it would be great.

 
5:27 AM
0
Q: Surface that contain a line which is not a principal direction

EnigmaSo I tried looking at level surfaces to find such an example, but I wasn't able to generate one. Could someone suggest some possible surfaces with this attribute.

0
Q: Question regarding Odd and Even function

Taylor TedLet $f:R \rightarrow R$. Define $g:R \rightarrow R$ by $g(x)=f(x)(f(x)+f(-x))$ Then which of following is/are correct? A. $g$ is even for all $f$ B.$g$ is odd for all $f$ C.$g$ is even if $f$ is even D.$g$ is even if $f$ is ofdd` Taking f=sinx eliminated option B.What if i take $f$ to be ...

Words such as question are uninformative in titles. Please edit the title so that it better describes the specifics of your question. Do not hesitate to make it longer or include a formula if needed. More tips here. (autocomment)Normal Human 21 secs ago
0
Q: Anything wrong with following analysis on $y=\tan(x+y)$?

ybtang21cGiven $y=\tan(x+y)$, we want to discuss the derivative of $y$ of $x$, says $y'_x$. Assume that $y$ is a function of $x$, then by $y(x)=\tan(x+y(x))$, the derivative of both sides remains equal, $$y'_x=y'_x\sec^2(x+y),\eqno{(1)}$$ then we get $y'_x=-\csc^2(x+y)$. But if we analysis equation (1)...

 
5:44 AM
0
Q: Bivariate Distribution

Venky I am trying to understand bivariate probability distribution functions and i am following All of Statistics: a concise course in statistical inference book In this book the author give one example for joint mass function as mentioned below: Suppose that f (x, y) = x + y if 0 ≤ x ≤ 1, 0 ≤ y ≤ 1 ...

 
6:01 AM
0
Q: Which interpolation method for complicated, smooth curves?

mavaviljWhich interpolation method should I use for complicated "smooth" curves such as $\frac{sin(x)}{x}$ for $x>0$.

0
Q: The Generalized Poincare-Bendixson Theorem2

MikeLet f be a $C^{1}$ vector field in an open set $E\subset$ $R^{2}$ containing an annular region A with a smooth boundary. Suppose f has no zero in $\bar{A}$ ( the closure of A) and f is transverse to the boundary of A, pointing inward. How can we prove that, if AA contains a finite number of cycl...

0
Q: Convolution of several functions

thalesI am trying to find the joint distribution of several exponential decay functions all of which have different decay rates.

 
6:17 AM
0
Q: Who were the biggest Fields medal snubs?

lobsterismOf people in their mid-20's who seemed like sure-fire future Fields medalists, who just never quite got to Fields level, and why didn't they succeed?

0
Q: Congruence class solutions of linear equations

CandiceI'm not sure how to find the congruence class solutions of linear equations or systems of linear equations. For example, how would you solve 3x + 4y = 4 (mod 6)?

Welcome to Math.SE, lobsterism. Tag (contest-math) should not be the only tag a question has. Please add a tag for a subject area to which the question belongs. (autocomment)Normal Human 39 secs ago
 
0
Q: Should we flag below comment recived on question

mk08I am a frequent user on Stack Overflow. I came across THIS question. The question itself is not asked with proper study, I agree. However, I think it is not appropriate to give comments which are tracking down User's personal history. Shouldn't we flag such comments or ask to delete? Adding a s...

 
6:37 AM
0
Q: Discrete graph theory

user296887Would anyone can explain detail for me this question or provide an example to verify the answer( regardless whether you think it does or it does not) Random d-regular graphs. Explain why the following claim holds: Claim: Any property that holds a.a.s. in the pairing model holds a.a.s. for random...

0
Q: Continuity and Cauchy sequences

RooneySo i'm revising for my final and i have encountered this problem that uses continuity and cauchy sequences. Let $f:D\rightarrow \mathbb{R}$ be continuous and let $(x_n)$ be a cauchy sequence. a) give an example to show that $(f(x_n))$ isn't Cauchy b) If D is compact, then show that $(f(x_n))$ ...

Words such as question do not add information to titles. Please edit the title so that it better describes the specifics of your question. Do not hesitate to make it longer or include a formula if needed. More tips here. (from a bot)Normal Human 51 secs ago
 
6:55 AM
0
Q: How to find triangle height if I know its area and angles

Abcdef XyzFor example, there is a triangle ABC with angles α = 45 degrees, β = 120 degrees, γ = 15 degrees. Area S = 15. How to find all three heights of the triangle?

0
Q: Discrete math question relate to graph theory

user296887Can someone explain help me how to solve this question in step by step so I can understand clear( since I tried but I could not figure out the answer) I appreciate for your time and help Consider a binomial random graph G ∈ G(n, p), where p = p(n) = (ln n + ln ln n + c)/2n for some constant c ∈...

Welcome to Math.SE, user296887. Words such as question are uninformative in titles. Please edit the title so that it better describes the specifics of your question. Do not hesitate to make it longer or include a formula if needed. This site uses MathJax formatting of formulas. More tips here. (autocomment)Normal Human 38 secs ago
0
Q: What's wrong with my solution to an inital value ODE problem?

SherryThe following is a problem from Weinberger textbook "A first course in partial differential equations". The answer key is $u=\log x$. I have spent three hours on this problem but still cannot reach the same result. The following is my solution. I checked it many times and it looks correct to ...

Title contains wrong, problem. Tagged differential-equations but mentions "partial". What's wrong with my solution to an inital value ODE problem?
0
Q: A related rates problem on distance increasing

LyndtA and B are initially standing together of a point P. A starts jogging north at a constant speed of $2.4m/s$. One minute later, B starts jogging east at $2.7m/s$. Three minutes after A started moving, at what rate is the distance between A and B increasing? I don't know how to start solving this...

 
7:26 AM
0
Q: Prove a certain set of complex numbers is open

GeorgeLet S be a set of all complex numbers z such that |z|>1. Show this set is open.

 
1
Q: Lose the excess [weight]

MagischInspired by this other burnination request, I sifted through some of the 367 Questions tagged with weight. In my opinion this constitutes an obvious meta tag and the tag adds almost no information to any question asked. The tag weight in my opinion fails following criteria: Does it describe...

 
0
Q: How to find the chord $CD$?

Aditya Agarwal So this is the question. And the answer is $10 \ cm$. I cannot see a way out. Is similarity to be used? Or there will be a construction? Please guide me.

Short title. Short question. Question contains please. How to find the chord $CD$?
 
7:45 AM
0
Q: Off-topic, really?

nooberCSS selectors for mosaic I wonder, since when is it that CSS questions are off-topic? Maybe, the guys just didn't get it?

 
0
Q: How do we multiply two qualitative values?

Tu PhamIs there any possible way that we could multiply two qualitative values, without quantifying these values? Please see the following link to see the an example. http://sbassoc.org/wp-content/uploads/2012/05/Gleicher-Change-Formula-Graphic.jpg

Short question. Question contains please. How do we multiply two qualitative values?
0
Q: Finding Maximum number of people that can be seated

user296897Maximum number of people that can be seated in n*m seats where n is number of rows and m is number of columns where some people uses left armrest(l),right armrest(r),both(b) or none(z).

 
8:00 AM
0
Q: 4-Cycle Implies 2-Cycle

JuliusHello everyone I'm having trouble with this question from my Dynamical Systems course: 4-periodic maps. Let f : [α, β] −→ [α, β] have an orbit of period four {a, b, c, d} where a < b < c < d. a) Enumerate all of the possible ways that f can act on a, b, c, d. For instance, f(a)=b, f(b)=c, f(c)=d...

 
8:39 AM
0
Q: How to solve the equation $\Delta \log \sqrt{E}=-\lambda^2 E$

Victor ChenHow to solve the equation $\Delta \log \sqrt{E}=-\lambda^2 E$, where $\lambda$ is a constant, and $E(u,v)$ is more than three times differentiable.

1
Q: Stuck on GEB chapter 9 - is b a MU number? is b a TNT number?

Tomáš DvořákI'm reading through Gödel, Escher, Bach, and I found myself stuck at chapter 9. I've been rereading through several times already, but I must be missing something. To clarify my background, I'm a computer scientist, not a mathematician. On page 273, D. R. Hofstadter states that Could it be, ...

0
Q: maximize flux integral

michaelLet S be the unit sphere. I am going to give you three fields, F1,F2,F3. For each field, you are allowed to choose a subset of the sphere, S1, S2, S3. Your goal is to make the flux integral as large as possible in each case. Then calculate each integral. a) F1 = e_r b) F2 = i. c) F3 = . Well,...

 
8:56 AM
0
Q: Finding the ditribution law

NehoraiThowing cube twice, Let $X$ be the sum of the two throwing Find the ditribution law My try $$ \begin{array}{c|lcr} &2&3&4&5&6&7&8&9&10&11&12 \\ \hline \text{P}_\text{X}(x) & 1/12 & 1/12 & 1/12&1/12&1/12&1/12&1/12&1/12&1/12&1/12&1/12 \\ \end{array} $$ But the sum shouldn't be $1$0? now th...

 
9:10 AM
0
Q: Let's do the bounty dance!

Martin - マーチンI have had this on my mind for quite some time already and I thought that the time before the year passes might be good to introduce this idea. Let's get some Bounties rolling... Bounties are a good way to draw attention to questions. There are various reasons, why one would want to award bount...

 
0
Q: Show that $\lim\limits_{z \to \infty} f(z)=0$ in |z|>1 where f is holomorphic.

LeilaIf f is holomorphic function in |z|>1 such that $ \lim\limits_{z \to \infty} \frac{Re(f(z))}{z} =0 $ show that $\lim\limits_{z \to \infty} f(z)=0$. When I first tried this question, I thought about Schwartz's Lemma, in which defines $g(z)=\frac{f(z)}{z}$ in |z|<1. Then when I thought it more car...

0
Q: What is P(Y1−Y2>3)?

TheValarsLet $Y_1 $and $Y_2$ have joint density function: $$f (y_1, y_2) = e^{-(y_1+y_2)}, \text{for all } y_1 >0,y_2 >0 $$ What is $P(Y_1−Y_2>3)$? My attempt: $$P(Y_1−Y_2>3) = P(Y_1−3> Y_2)$$ $$\int_{0}^{\infty}\int_{0}^{y_1-3} e^{-y_1-y_2} dy_2dy_1$$ Evaluating the integral, the value returns: $1-...

 
9:35 AM
0
Q: Does the dual space of W^{1,2}(B) contain L^1(B)

van abelSuppose that $B\subset R^m$, $m\geq 2$, be a unit ball, and $W^{1,2}(B )$ be the Sobolev space. My quesion is does $(W^{1,2}(B))^*\supset L^1(B)$?

0
Q: simple random walk problem and its expected value

AarONGAragadA man sat in a round table of 10 seats by himself and put his backpack in a seat next to him. In the end, He got drunk and did not remember which seat did he put his bag. He decides to find it. But he is so drunk that at each step he moves to one of the two adjacent seats with equal probability. ...

0
Q: A Doubt in Davenport's Multiplicative Number Theory

Yash V. SinghIn his text Multiplicative Number Theory on page 9, Davenport mentions that another means of expanding the L-function is known and then mentions the fact that, $$ \mathcal{F} \sum_{n=1,n \; odd} \frac{1}{n}e_q(mn) = \begin{cases} \frac{1}{4} \pi & if \; 0<m<q/2 \\ -\frac{1}{4} \pi & if \; q/2<m<...

0
Q: Limit of relation sums of exponential functions

Алексей ДанчинHelp to solve example: $$ \lim_{n\rightarrow\infty}\frac{\sqrt[3]{7^{3n-1}+3^n}}{3^n+4*7^{n+1}} $$

Words such as doubt are uninformative in titles. Please edit the title so that it better describes the specifics of your question. Do not hesitate to make it longer or include a formula if needed. More tips here. (autocomment)Normal Human 1 min ago
-1
Q: How many numbers between 29 and 39?

SaiWe would like to know how many numbers between 29 and 39. Is it 9 or 10. Thanks.

0
Q: Sum of all values of $b$ if the difference between the largest and smallest values of the function $f(x)=x^2-2bx+1$ in the segment $[0,1]$ is $4$

Vinod Kumar PuniaFind sum of all possible values of the parameter $b$ if the difference between the largest and smallest values of the function $f(x)=x^2-2bx+1$ in the segment $[0,1]$ is $4$. I found that the smallest value of $f(x)=x^2-2bx+1$ is $1-b^2$ But i do not know what will be the largest value of the ...

Question contains please. [Sum of all values of $b$ if the difference between the largest and smallest values of the function $f(x)=x^2-2bx+1$ in the segment $[0,1]$ is $4$](math.stackexchange.com/q/1565576)
 
9:54 AM
0
Q: Visual Studio Interactive Tags

Obsidian PhoenixWhilst asking this question, I created a new tag: C#-Interactive - in the same vein as the existing F#-Interactive. In the efforts to keep the tags "correct", I wanted to bring this up here for discussion. In my view, there are a couple of different tag structures that could be used: Use sepa...

 
10:08 AM
0
Q: Prove that if the sequence $(f_n)$ converges to $f$, then $f$ is increasing

luka5z Let $(f_n)$ be a sequence of increasing functions from $[a,b]$ to $\mathbb{R}$ and $f$ be its pointwise limit. Choose $x$ and $y$ such that $x\ge y$. There are $\epsilon>0$ and $N,M$ such that $|f_n(x)-f(x)|<\epsilon$ and $|f_m(y)-f(y)|<\epsilon$ whenever $n>N$ and $m>M$. If we choose $k>\mi...

0
Q: Test to find the number of primes below X

user43087So I was wondering if this idea for finding the number of primes below X could work and whether its been tried before: Its slightly optimistic as it suggests that from 7 primes (2,3,5,7,11,13 and 17) and there combinations we should be able to see how many primes there are below 4,594,590 (2*3*5...

0
Q: proof or deproof linear equivalence of X, X is an amount.

RayofCommand Again I am stuck at some proof. I need to proof or deproof that for all linear equivalences: R:(X,X) is R = So far I think it is correct because we get symmetry and linearity, but I have troubles to proof it. Any help is upvoted immediately.

This site uses MathJax formatting of formulas. More tips here. (autocomment)Normal Human 32 secs ago
 
10:23 AM
0
Q: Existence of solution of $\frac{\partial f}{\partial t}=-\Delta f+|\nabla f|^2-R(x,t)$

lanse7ptyWhen $t=t_0$, $f(x,t)=f_0(x)\in L^2(U)$. $t\in [0,t_0]$ and $U$ is a open subset of $R^n$.$R(x,t)$ is bounded and smooth about $x$ and $t$. I don't whether suitable the conditions is ,if not, please correct it . How to show the existence of solution of $\frac{\partial f}{\partial t}=-\Delta f...

Question contains please. Tagged pde, differential-equations. Tagged differential-equations but mentions "partial". Existence of solution of $\frac{\partial f}{\partial t}=-\Delta f+|\nabla f|^2-R(x,t)$
0
Q: Estimate $\sum_{\left|\frac{k}{n}-x\right|>\delta}\binom nk x^k(1-x)^{n-k}$

LauraHow to establish the following inequality? $$\sum_{\left|\frac{k}{n}-x\right|>\delta}\binom nk x^k(1-x)^{n-k}\leq \frac{x(1-x)}{n\delta^2}$$ where $\delta >0$.

0
Q: For $f(x) \in \mathcal{F}_{q^n}[x]$, how to find all $p(x)f(x)$ s.t. its coefficients are $\mathcal{F}_q$-linear over coefficients of $f(x)$?

Piotr SemenovMostly, I am interesting how to find the non-trivial $p(x)$ having the coefficients from $\mathcal{F}_{q^n}$ rather from $\mathcal{F}_q$.

Short question. [For $f(x) \in \mathcal{F}_{q^n}[x]$, how to find all $p(x)f(x)$ s.t. its coefficients are $\mathcal{F}_q$-linear over coefficients of $f(x)$?](math.stackexchange.com/q/1565594)
0
Q: What is theta value?

blue-skyUsing a pre-defined formula in Desmos android app the following example is given : What is value of theta used within formula for evaluating r ? Is it some implicit value, as it's value is not displayed.

Short title. What is theta value?
0
Q: Some sequence convergence problems

diogenesProve that any sequence {$x_n$} that satisfies $|x_n-x_{n+1}|<=\frac{1}{2^n},n\in\Bbb{N}$ is convergent. Prove that a sequence that satisfies $x_{n+1}-x_n\to0$ is not necessarily convergent.

Title contains problem. Some sequence convergence problems
-1
Q: Sign into the Sage Notebook v6.9

Angel BlascoActually I'm trying to acces into the Sage Notebook last version (6.9). But the system doesn't recognize my username or password. I tried it using my sagemathcloud user, my system user and my root user but always the same answer appears: "username is not in the system". How can I accesss into Sag...

Questions tend to get more attention when they have a tag for a broad area of mathematics relevant to the question. Some of these tags might fit. (autocomment)Normal Human 42 secs ago
 
10:52 AM
1
Q: Congrats on 1m [javascript] questions!

CerbrusI thought it'd be nice to write up a celebratory post, in honor of javascript passing 1,000,000 questions! My thanks go out the the massive community active in the tag, asking, answering and otherwise contributing to all kinds of interesting questions. On to 2M!

 
0
Q: How to get mathematical model from a data set with MATLAB

AndreaFI have some values imported from excel about the annual sell of a product, with 3 var, month, price, sold items I have seen how to plot a chart using curve fitting tool but I need to define a math expression that can describe the relation between these variables. How can I do this in matlab? Is t...

0
Q: Sequences and series natural numbers

Atharvawhat is the sum of all three digit natural numbers that are multiples of 14, but not 21? What is a quick way of doing sums like these, as i cannot rely on intuition during timed exams

0
Q: Calculus help about modifying a function

user887912I am having trouble about modifying a function. We have a function that is believed to define a physical condition truly. However, according to my experiments the given function is insufficient. I want to modify the function in a way that it approximately fits to the experimental data. The origin...

Words such as help are uninformative in titles. Please edit the title so that it better describes the specifics of your question. Do not hesitate to make it longer or include a formula if needed. More tips here. (autocomment)Normal Human 26 secs ago
0
Q: f(x) = x^3 + 3px + q

user296928I'm having trouble solving this. Investigate the extreme values of the function f, defined by f(x) = x^3 + 3px + q, for all x ∈ R, p, q being fixed real numbers.

Welcome to Math.SE, user296928. Consider replacing (analysis) with a more specific tag for the relevant branch of analysis. (autocomment)Normal Human 29 secs ago
0
Q: Find all natural numbers n such that ($\phi$)=12

ematth7I need help finding all natural numbers n such that ($\phi$)=12. My professor told us to use $\phi$(mn)=$\phi$(m)$\phi$(n). I know that 12 can be factored into 3,4 12,1 6,2.

 
11:11 AM
0
Q: Cannot ask for close, error searching for duplicates

chqrlieI tried to close question Why is scanf() causing loops to continue without waiting for input? as a duplicate of question Why is scanf() causing infinite loop in this code? but the popup window shows a generic error preventing submission.

 
0
Q: Nice fractals which are computationally intense.

mathreadlerOh hello guys. I am in the middle of challenging myself to putting my computer and math skills together, trying to build a small hobby computational cluster. Being interested in fractals for a long time I have been able to calculate silly amounts of Mandelbrot pixels really fast in my new playgr...

Words such as nice are uninformative in titles. Please edit the title so that it better describes the specifics of your question. Do not hesitate to make it longer or include a formula if needed. More tips here. (autocomment)Normal Human 21 secs ago
 
11:25 AM
0
Q: A simple bound implied by a moment condition

Jie WeiSuppose $Y_{i}$ are i.i.d., and for some $\theta > 5/2$, $E(|Y|^{\theta})<\infty$. How does this condition imply that $sup_{1 \leq i\leq n} |Y_{i}| = o_{P}(n^{2/5})$?

0
Q: Limit with -inf result, but how?

Алексей ДанчинWhat do with this: $$ \lim_{n\rightarrow\infty}\frac{1}{\sqrt{n}(n-\sqrt[3]{n^3+2})} $$ Wolfram say -inf, but how?

Consider adding a tag for a broader subject area to which the question belongs. Some of these tags might fit. (from a bot)Normal Human 40 secs ago
0
Q: Ricci soliton and Ricci flow

lanse7ptyWhen I read the Ricci Soliton geometric meaning, I get stuck in the plugging in the Ricci flow as picture below.I don't know how to plug in it,in my opinion, Ricci flow is $\partial_tg_{ij}=-2R_{ij}$. Whether it mean $\mathcal L_{X_P}g_{ij}+2R_{ij}+2\lambda g_{ij}=0$ ?If so , how to get Ricci sol...

0
Q: How to Prove Triangle Centers in Tetrahedra

halcyon_kHow would you prove the existence of triangle centers in tetrahedra, for example, the incenter, circumcenter, or centroid?

Tag (proof-writing) should not be the only tag a question has. Please add a tag for a subject area to which the question belongs. (autocomment)Normal Human 27 secs ago
 
0
Q: Trailing backslash in strings breaks code formatting

sodawillowI did not find an answer to this after searching a bit : On stackoverflow.com, is there a way to avoid breaking a code block's formatting when including a string with a trailing backslash ? It's common on the Windows platform to use strings like C:\Users\me\. If you include this in a code block...

 
0
Q: On normalized error measures

Eric S.I have function values $f_1,\ldots,f_n$ that are approximated by data $y_1,\ldots,y_n$. I am looking for a measure that describes the error in the data $y_1,\ldots,y_n$ and I want the measure to take values between $0$ and $1$. I am familiar with the Root Mean Squared Error (RMSE): $$\mathrm{RM...

0
Q: Unitary transformation

12saI have a matrix in following form $$A=\begin{bmatrix} 1&0&0&0&0&0&0&0\\ 0&-1&0&0&0&0&0&0\\ 0&0&1&0&0&0&0&0\\ 0&0&0&-1&0&0&0&0\\ 0&0&0&0&1&0&0&0\\ 0&0&0&0&0&-1&0&0\\ 0&0&0&0&0&0&1&0\\ 0&0&0&0&0&0&0&-1\\ \end{bmatrix}$$ I need some help to reduce the matrix in following form using unitary transfo...

 
11:45 AM
0
Q: Are text editor regex questions off topic?

dan1111This question, which asked about the behavior of a regex in Notepad++, was recently closed as off topic as a question about "general computing hardware and software". This doesn't seem right to me: regexes are a programming language, so shouldn't a question about regexes in any domain be on topi...

 
0
Q: Is conditional density function integrable?

unknownWhat are conditions of conditional density function being integrable?

0
Q: finding residues for poles

AlanI'm struggling to find the residues of the equation -zln(z)/((z^2+a^2)(2-z)) with poles at z=+/-ai and z=2 I have the residue for z=2 as -2ln2/4+a^2 but I am struggling to find the residues for +/-ai can anyone help me?

0
Q: Can anyone tell me

Tanishq VarshneyThe fourier series for $$ \ln (2 \sin x/2)=\sum_{k=0} \frac{\cos kx}{k} $$ Is the fourier series for $$ \ln ( 2 \cos x/2) $$ same ???

Words such as anyone do not add information to titles. Please edit the title so that it better describes the specifics of your question. Do not hesitate to make it longer or include a formula if needed. More tips here. (from a bot)Normal Human 29 secs ago
 
12:16 PM
0
Q: Projecting data onto a vector

TestGuestI have learned that projecting a vector a onto a vector b is done by multiplying the orthogonal projection of a (say $\mathbf{a_o}$) with the unit vector $\mathbf{\hat{b}}$ in the direction of b = Orthogonal projection of a onto b = $\mathbf{a_o}$ * $\mathbf{\hat{b}}$ However in the context ...

0
Q: Markov Chain Question. Show vector of stopped processes are i.i.d.

measureLet $X_n$ be a Markov Chain with a countable state space $S$, and supposed that the state $s\in S$ is recurrent. Denote $\tau_s^(k) = \inf\{n\geq\tau_s^(k+1) \, \vert \, X_n = s\} for $k\geq 1$. Also let $W_k = \tau_s^(k) - \tau_s^(k-1) Show that for $X_0$ = s, the vectors Z_k = (W_k, X_{\tau_s...

0
Q: A Poisson process game

elDin0An interesting puzzle I came across: For $T>1$, observe a Poisson process of rate $1$ on the time interval $(0,T)$. Every time we observe a point, we may choose to stop. To win the game, we must stop on the last point before time $T$. Else, if we stop at $t$ and there is another point in $(t,...

0
Q: Is it true that is $P^n=<a^n>$ such that n is a positive integer?

user275240Let $P$ be an ideal of a ring R .Let $P$ be generated by an element $a$ (i.e. $P=<a>$). Is it true that is $P^n=<a^n>$ such that n is a positive integer? How could I prove that?

0
Q: Communicating classes property

matiskayLet $i \neq j$. Proof that $i \leftrightarrow j$ if and only if there are $m, n \geq 0$ such that $\mathbb{P}(X_{m + n} = i, X_{m} = j \mid X_{0} = i)$.

Short title. Short question. Communicating classes property
0
Q: Determine all c such that $a_0=0 ; a_{n+1}=\frac{a_n^2+c}{2}$ converges

H.T.HDertemine all c such that the recursive sequence ${a_n}$ defined by setting $a_0=0 ; a_{n+1}=\dfrac{a_n^2+c}{2}$ converges.

0
Q: Group of order 255 is cyclic

user289143Let $G$ a group and its order is $255$. Prove that $G$ is cyclic. I easily demonstrated that the group has only one $Sylow 17-subgroup$ $P$ that is normal in $G$ and it's cyclic since it is of a prime order. Then $G/P$ is also cyclic since a group of order $15$ is cyclic. Then $G$ can be seen as...

0
Q: solving a geometry problem for middle school children

Florin M.On the plane of the $ABE$ $triangle$ we raise $CB$ as perpendicular. $M$ is the middle of $AB$. $AB=8cm$ and $2*CB=AB$. Requirements: a) Calculate the distance between the points $C$ and $B$. b) Show that $EM$ is perpendicular on the plane $(ABC)$ c) Consider $D$ such as $ABCD$ is a rectangle...

 
1:09 PM
0
Q: Simplify this expression with cos and sin

prishila$$cos12x/cos4x-sin12x/sin4x=(cos^2 6x-sin^2 6x)/(cos^2 2x-sin^2 2x)-(2sin6xcos6x)/(2sin2xcos2x)$$ What should I do next?

0
Q: Prove $tgh(\frac{x}{2})=(\frac{cosh(x)-1}{sinh(x)}$

gbox Prove $tgh(\frac{x}{2})=\frac{cosh(x)-1}{sinh(x)}$ I have started with: $tgh(\frac{x}{2})=\sqrt{tgh^2(\frac{x}{2})}=\sqrt{1-cosh^{-2}(\frac{x}{2})}=\sqrt{cosh^2{\frac{x}{2}}-sinh^2{\frac{x}{2}}-cosh^{-2}(\frac{x}{2})}$ I am stuck here

 
0
Q: Reputation gained since last visit to reputation tab is wrong

ablighPer this screenshot I have 15,003 reputation, and have gained 4k since some point in time. Hovering over it says this has been since 'your last visit to the reputation tab'. That's wrong, because I visited it yesterday, and I haven't gained 4k reputation in a day (insert mandatory Jon Skeet ref...

 
1:26 PM
0
Q: Integrating $\int_{-1}^{1}\frac{dx}{(x-a)\sqrt{1-x^2}}$

LumiormI'm asked to find the value of $$\int_{-1}^{1}\frac{dx}{(x-a)\sqrt{1-x^2}}$$ where $a$ is complex and $a\not\in[-1, 1]$. I think I should use Cauchy's integration formula but don't know how to apply it.

0
Q: Reflection in a plane

Anirudh GangwalLet P be the plane in $R^3$ defined by $x + 2y + 2z = 0$. Projection of any vector u on this plane is defined by Au = proj $_P$ u. A = $$ \begin{matrix} 8/9 & -2/9 & -2/9 \\ -2/9 & 5/9 & -4/9 \\ -2/0 & -4/9 & 1/9 \\ \end{matrix} $$ Let u be any vector in...

 
1:42 PM
0
Q: Exact Simplification of Meijer G function

VicI am trying to simplify the following Meijer-G funtion \begin{equation} G^{2,2}_{2,2}\Bigl({}^{0,\, 1-m}_{0,\,0}\Bigr |x) \end{equation} But the Matlab(MuPAD) and WolframAlpha give me different results, which are not equivalent (The second terms of both answers are equivalent, but not the first t...

Tagged proof-verification. Exact Simplification of Meijer G function
0
Q: Re-arranging exponential equation

Michael RobertsNot quite applicable to many situations, but I'm extending this as more of a "I need help/tips" on re-arranging an equation. I something like this: $$ 0.02=\frac{1}{(1+c)}\left(1-e^{-\frac{Z}{y/(1+c))}}\right) $$ I want to re-arranging this equation for $c$. How would this be done? I know that ...

0
Q: difficulty in solving PDE.

zafranGiven that $yu_x+xu_y=xy$ , $x\geqslant0$, $y\geqslant0$ with $u(0,y)=e^{-y^2}$ for $y>0$ , and $u(x,0)=e^{-x^2}$ for $x>0$. After solving this I got $u(x,y)=\ln (x)+f(\frac{ { y }} x ) $ My doubt is that how to use initial values in this case because denominator will become zero? Or is ...

 
2
Q: Uninstall [uninstall]

blackI think the uninstall should be removed: all the highest-rated questions are clearly off-topic and they date back to a couple of years ago. It should be rather painless, with just ~1200 questions tagged with it.

0
Q: Regex to find unescaped html tags in Stack Overflow posts

Mr ListerI've been looking at "hidden messages" in posts lately, where the question reads, for instance Should I use or ? and the real question becomes visible only when editing: Should I use <span> or <div>? Of course it's not difficult to add backticks to this line manually, but when a very lon...

 
0
Q: Find the product xy.

Mathematics Find product $ xy $ if both x and y are real. After applying basic log identities, I tried equating value of $ 1/(log x)^2+1/(log y)^2 $ but I am not getting any fruitful result.

Short title. Find the product xy.
0
Q: Invariant measure and invariant distribution for markov chain

Lama Al SayeghI am so lost between the measure and distribution! I do know the definition for each but I couldn't apply it for this question : Comment on the existence and uniqueness of invariant measure and of invariant distribution: Does an invariant measure always exist? Is it unique? Does and invariant dis...

Welcome to Math.SE, Lama Al Sayegh. This site uses MathJax formatting of formulas. More tips here. (autocomment)Normal Human 21 secs ago
0
Q: Simple inequalities question

CAFGiven that $0<a<-b$ what may we deduce about $(a+b)/a$? This implies that $(a+b)/a < 0$ which I can write as $-|a+b|/|a| < 0$ so that $|a+b|/|a|>0$. Is this all I can conclude? My prof says to me that the quantity $|(a+b)/a| < 1$ but I haven't been able to see this yet. Thanks!

Words such as question do not add information to titles. Please edit the title so that it better describes the specifics of your question. Do not hesitate to make it longer or include a formula if needed. More tips here. (autocomment)Normal Human 20 secs ago
 
2:02 PM
0
Q: Is it "not done" to serial- edit your own posts?

Jacob VlijmWhat I ran into Strolling through the keys in dconf-editor, looking for something else, I ran into the screen's information in /org/compiz/profiles/unity/plugins/core/outputs which showed the values: ['1680x1050+0+0', '1280x1024+1680+0'] on my system. This surprised me a bit. I use to ge...

1
Q: Lets turn the blinking off ... on .. off.. on ... PERMANTENTLY OFF

HennesThe blinking tag is used in 18 questions. Mostly about blinking LEDs, cursors and minitors which turn on and off. I suggest we permanently blink it out of exitence.

 
0
Q: Finding an algorithm for the given problem - how should AI proceed?

Adam BaranyaiLet's presume that we have a PVP fight scene, where 1 or 2 heroes, are fighting 3 monsters. The monsters that the heroes are fighting are the following: Skeleton (2 of this monster) Health: 1 Defense: 0 Attack: 1 Death Knight (1 of this monster) Health:...

Welcome to Math.SE, Adam Baranyai. This site uses MathJax formatting of formulas. More tips here. (from a bot)Normal Human 21 secs ago
0
Q: Examples of Partial Combinatory Algebras with surjective pairing?

Malice VidrineWhat are some good examples of partial combinatory algebras (a.k.a. Schoenfinkel algebras) with surjective pairing? I mean this in the sense that, if $\mathsf{D}$ is the pairing combinator and $\pi_0,\pi_1$ the projection combinators, then $\mathsf{D}(\pi_0x)(\pi_1x)=x$ for all $x$. Especially in...

Consider adding a tag for a broader subject area to which the question belongs. Some of these tags might fit. (autocomment)Normal Human 21 secs ago
0
Q: Drwaring the phase portrait

SalamoLet $\lambda >0$. Consider the non-linear system $$ \begin{cases}\dot{x}=x^2\\\dot{y}=-\lambda y\end{cases}. $$ The aim is to get an idea of the phase portrait. There are some things I can see directly: First of all, $(0,0)$ is an equilibrium. On the $x$-axis, we are moving to the right. On t...

0
Q: Help me to find all posibble values of a,b,c

AyushakjIf a^2=bc+1 and b^2=ac+1.How do I find all possible values of a,b,c.

Welcome to Math.SE, Ayushakj. Words such as help are uninformative in titles. Please edit the title so that it better describes the specifics of your question. Do not hesitate to make it longer or include a formula if needed. More tips here. (autocomment)Normal Human 21 secs ago
0
Q: Question about Sobolev spaces on manifolds

YamamotoLet $(M,g)$ be a manifold with metric $g$ parametrized by the mapping operator $S$ and parametric domain $\Omega$. The sobolev space of order one with respect to the $L_2(M)$-norm $H^1_2(M)$ is defined as follows: Let $C_2^1 = \{ u \in C^\infty(M) s.t. \forall j = 0,1: \int_M |\nabla^j u |^2 dv(g) <

Words such as question do not add information to titles. Please edit the title so that it better describes the specifics of your question. Do not hesitate to make it longer or include a formula if needed. More tips here. (from a bot)Normal Human 21 secs ago
 
2:27 PM
-6
Q: Pay for good answers

BoundlessWhen I posted a question to SO a year or two ago there would be an overabundance of answers. Now I'll post a question and days pass with less than 10 views, and 0 answers. I would be willing to donate via PayPal or some other source to a fund that I'd use to pay for good answers. I would imagine ...

 
0
Q: Invariant under isometric

lanse7ptyI am not familiar with the formal compute about the invariant under diffeomorphism (isometric),so I want a detail example. For example,$M,N$ are Riemannian manifolds, $\Phi :M\rightarrow N$ is diffeomorphism.How to formally show $$ \int_MR(\Phi^*g_{ij}) dV(\Phi^*g_{ij})=\int_N R(g_{ij})dV(g_{ij}...

0
Q: Is there any easy (intuitive) way to apply a linear transformation to these types of trajectories?

YoTengoUnLCDI'm studying systems of differential equations an their representations via phase portraits at the moment, I was taught that when sketching the phase plane of a $2\times2$ system of the form: $$ \dot X=AX $$ With solutions $X=c_1\Re (e^{(\alpha+i\beta)t}\xi_1)+c_2\Im(e^{(\alpha+\beta i)t}\xi_1)...

0
Q: On $AX=XB$ matrix form

TurboGiven $A,B\in\Bbb R^{n\times n}$ when is there a real matrix $X$ such that $$AX=XB$$ holds? Is there such a matrix for any pair of $(A,B)$?

Short title. Short question. On $AX=XB$ matrix form
0
Q: Smith Normal Form and free basis

jelenaIf I reduce a matrix to Smith Normal Form, how do I find the free basis of the smith normal form of the matrix?

0
Q: Group transitivity of alternating group $A_n \quad n>2$?

AnnaStenDoes it not suffice to point out that $$(i, k)(i, j)\in A_n$$ The element at location $i$ is mapped to the element at location $j$ and and the element at location $j$ is mapped to some third location $k$. Does this not prove transitivity if $n>2$?

0
Q: Multiplying by 2pi

DawgI am confused on how to multiply by pi using a calculator(ti-84). I am working in radians. Convert 2.51 x 10^4 rad/min : 2.51 x 10^4 * 2pi divided by 60 My answer was: 2628.465854 rad/s The tutorial video I was watching answer was: 2627.1 rad/s Which answer is correct? Thanks.

Short title. Multiplying by 2pi
0
Q: find the derivative of the definite integral from 0 to ln(x) of $Sqrt((t^5)+1)dt$

Ilia LabkovskyI'm not sure how to take the derivative of a definite integral.

0
Q: Prove various statements regarding linear maps

luka5z Let $V$, $V'$ be normed vector spaces and $f:V\rightarrow V'$ be a linear map. I would be very grateful if someone could verify if my proofs of the below statements are 100% correct. Also, I can't prove the first half of b). a) If $f$ is continuous at one point, it is continuous everyw...

0
Q: Bivariate Normal distribution problem.

User9523The exponent $e$ of a bivariate normal density is given as follows : $$ -\dfrac{1}{102} [ (x+2)^{2} - 2.8(x+2)(y-1) + 4 (y-1)^{2} ]$$ We need to find : E(X) , E(Y) , Var(X) , Var(y) and $\rho$ (correlation coefficient). I tried to compare that $102$ with $ 2 ( \sqrt{1 - \rho^{2}})^{2}$ , but t...

 
3:04 PM
0
Q: Removing [ecmascript-6] tags from questions

RGrahamI've seen a number of questions recently where the ecmascript-6 tag has been removed from a question. I don't really want to name & shame anyone so I'll not post the links to the questions. These mainly appear to be questions where the root cause of the problem was not due to ES6 syntax - but a ...

3
Q: Do we need [project-design]?

JALI was recently reviewing suggested edits when a wiki except suggestion came up for the project-design tag. Not seeing how the tag or its description would be useful, I rejected the edit. Looking further, I see 19 questions with the project-design tag. I feel as if this tag is too broad for Sta...

 
0
Q: If $P(x)=p_nx^n+p_{n-1}x^{n-1}+...+p_o$ is divided by $(x-a)$, show that the remainder is $P(a)$. Need help verifying my proof.

shredalertIf $P(x)=p_nx^n+p_{n-1}x^{n-1}+...+p_o$ is divided by $(x-a)$, show that the remainder is $P(a)$. Need help verifying my proof. Proof: If $P(x)$ is divided by $(x-a)$, it gives a quotient of $Q(x)$ and remainder $R$. $P(x)\equiv(x-a)Q(x)+R$ Substituting $a$ for $x$ in this identity gives $R$. ...

Tag (proof-verification) should not be the only tag a question has. Please add a tag for a subject area to which the question belongs. (from a bot)Normal Human 21 secs ago
0
Q: $p=k n+1$ prime

34FFT ConvThis is a part of the question from CLRS (Introduction to Algorithms) Chapter on FFT and Polynomials. I am self reading and am stuck at this part. Let $n$ be a power of 2. Suppose that we search for the smallest $k$ such that $p = kn + 1$ is prime. Give a simple heuristic argument why we might ...

Short title. $p=k n+1$ prime
0
Q: Joint probability problem of X and Y

MartinCould anyone help me with this question? Thanks! Suppose that two balls are randomly chosen without replacement from an urn containing $5$ white balls and $8$ red balls. Let the random variable X equals $1$ if the first ball chosen is white and equals $0$ the first ball chosen is red. Let the ra...

Title contains problem. Joint probability problem of X and Y
0
Q: Taylor polynomial p2(x)

user296523Find P2(x) for f(x) = e^cosx expanded about x0 = 0 and then I have to find M > 0 so that |f(x) - p2(x)| <= M|x|^3 for all x ∈ R. I think p2(x) = ex but I'm not really sure. And I really need help with the second part, where I have to find M.

 
3:24 PM
0
Q: Lets nuke a tag down to its origin

HennesWe have have five questions tagged origins. Three are about origins of something (history). One is about the screen positioning (with origin at 0,0) and one is about a game client. I propose to be unoriginal in our nuking.

0
Q: Few users with equal amount in a bounty question

Igor TyulkanovRecently I've answered a bounty question. After 7 days period answers was not awarded. When grace period ended I and other user had an equal amount. Other user was awarded (half bounty amount) by system. So I'm just curious: by what criteria the system selects the winner, if few users have the sa...

 
0
Q: What is some simple to prove very counter-intuitive result obtained by Choice?

YoTengoUnLCDI'm aware of some theorems like the Banach-Tarski's which yield very counter-intuitive results, however, it's proof is far beyond my knowledge, so I'm looking for some result that is easy to prove and gives a counter-intuitive result, using Choice. Does anyone know any examples?

0
Q: Why does RSA fail when p=q

Jean LilleA lot of questions about this have unsatisfying answers that either argues how unsafe RSA is when $p=q$ or points out that $\phi(n)$ is not $(p-1)(q-1)$ when $p=q$ and that $\phi(n)$ must be used to make RSA secure. However, I'd like to know why the RSA fails, i.e why the Decipher $R$ is not equ...

0
Q: Find local max local min for f(x) = x^2 + 1/x

DreamerFind local max local min for f(x) = x^2 + 1/x, I tried to take first order derivative $$ f'(x) = 2x -x^{-2}$$ but how do I go from here.

0
Q: question on geometry circle

AyushakjThe figure below shows a broken piece of a circular plate made of glass.

Words such as question are uninformative in titles. Please edit the title so that it better describes the specifics of your question. Do not hesitate to make it longer or include a formula if needed. More tips here. (autocomment)Normal Human 21 secs ago
0
Q: Proof of translation property f(x), f(x-a)

Aaron ShifmanSo the subject of translations in a very simple case came up the other day, where for $x,y\in R$ $f(x-a)$ is just $f(x)$ translated to the right. This is trivial to show in a very hand waving sort of way: the function $f$ "looks back", and the fact that $f(x-a)$ goes to the right and $f(x+a)$ goe...

Tag (transformation) should not be the only tag a question has. Please add a tag for a subject area to which the question belongs. (from a bot)Normal Human 21 secs ago
0
Q: Let $u$ and $v$ be coprime elements of a principal ideal domain $R$. Prove that $R/(uv) \cong R/(u) \bigoplusR/(v)$.

AZ2015I found a similar exercise but I couldn't solve it also let $a\in R$ which is PID if $a=\prod_{i=1}^{k} p_{i}^{n^{i}}$ then $R/(a) \cong \bigoplus R/(p_{i}^{n^{i}})$

 
3:47 PM
0
Q: help for solving recurrence relation

user123I am trying to solve this recurrence relation $f(x)$ = $f(x - 3)$ + $\frac{(2(x - 3)^3 + 3(x - 3)^2 - 2(x - 3))}{24}$ but I don't know how to proceed further. So can some one please help me in solving this recurrence relation. And also please give me some hints on how to solve these kind of pro...

Welcome to Math.SE, user123. Words such as help are uninformative in titles. Please edit the title so that it better describes the specifics of your question. Do not hesitate to make it longer or include a formula if needed. More tips here. (from a bot)Normal Human 21 secs ago
0
Q: Accumulation Point Proof

samsoniteLet $D \subset R$ and let $a \in D$. Show that acc(D \ {a}) = acc(D). I'm not sure I really understand what this question is asking. How can I show that these accumulation points are equivalent?

0
Q: Let $P(x)=(m^2+4m+5)x^2-4x+7,m\in R$.If $3\leq x\leq 5$,then find the minimum of the minimum value of $P(x).$

Vinod Kumar PuniaLet $P(x)=(m^2+4m+5)x^2-4x+7,m\in R$.If $3\leq x\leq 5$,then find the minimum of the minimum value of $P(x).$ The minimum value of $P(x)=(m^2+4m+5)x^2-4x+7$ occurs at $x=\frac{2}{m^2+4m+5}$ So the minimum value of the $P(x)$ is $\frac{-4}{m^2+4m+5}+7$ But i dont know how to find the minimum v...

0
Q: Primitiv matrix

BaloownA matrix $M\in\mathcal{M}_n(\mathbb{R})$ is said to be primitive if it exists an integer $k$ such as $M^k$ has all its coefficients strictly positive. My question is : If $M$ is primitive and if $N$ is similar to $M,$ say $N=PMP^{-1}$ with $P\in\mathrm{GL}_n(\mathbb{R}),$ do we have that $N$...

Short title. Primitiv matrix
0
Q: Financial math for high school seniors

RErI'm giving a lecture to senior year high school students. I want to talk about the importance of mathematics and illustrate my point with some practical examples. I've decided to concentrate on financial mathematics. I've got about 20min to speak about math. I thought about explaining some basic...

Tag (soft-question) should not be the only tag a question has. Please add a tag for a subject area to which the question belongs. (autocomment)Normal Human 21 secs ago
0
Q: Why is 3.3 greater than 1/3?

iProgramI was thinking to my self one day, whats bigger, 3.3 or 3.3333333 (1/3)? I decided to write a program in swift to test it var threeDotThreeThree:Double = 3.3 var threeRequring:Double = 1/3 threeDotThreeThree > threeRequring The result of the line threeDotThreeThree > threeRequring was true. ...

0
Q: Some problem of Integration Theory

Alexei0709Need some help with this problems: *1. Conjecture the value of the limit $$ \lim_{n \to \infty}\,\int_0^{\infty}\,\left( 1+\dfrac{x}{n} \right)^ne^{-2x}\,dx $$ Prove your conjecture. Justify.* 2. Suppose the $f_n \in L^1(\mathbb{T})$, $n = 1,2,...$ and $\| f_n - f \|_{L^1(\mathbb{T})} \xright...

Title contains problem. Some problem of Integration Theory
0
Q: Is it ok to make an operation over some definition at very the first time you are defining it?

IntegralSuppose we have an expression and want to define it using some symbol. One can write something like "$X:= \text{Expression}$" to mean the symbol $X$ is defined to be the expression after the symbol "$:=$". From my understand, before the definition $X$ is supposed to be just a symbol, after the d...

 
00:00 - 16:0016:00 - 00:00

« first day (36 days earlier)      last day (538 days later) »